Égalité cardinale

Purée, je bloque encore sur une trivialité.
Soit $\kappa$ un cardinal infini.
À quelle condition (suffisante) peut-on affirmer que $\kappa^{< \kappa} = \kappa$ ?
Il me semble que "régulier" devrait marcher, mais je n'ai plus l'argument sous la main...

Réponses

  • Bon, je me suis mal exprimé. Il est clair que "inaccessible" suffit.
    Mais "régulier" ne suffit pas, car par exemple il n'y a aucune raison pour que $\aleph_1^{\aleph_0}$ soit égal à $\aleph_1$.
    Ma question est donc : y a-t-il une condition suffisante moins drastique que "inaccessible" ?
  • De mon téléphone je ne connais pas de mot consacré célèbre pour cette propriété.
    Aide les autres comme toi-même car ils sont toi, ils sont vraiment toi
  • Régulier ne suffit pas, voici l'argument naïf qu'on a en tête et qui bloque : $\kappa^{<\kappa} = \sum_{\mu <\kappa}\kappa^\mu = \sum_{\mu<\kappa, \lambda < \kappa} \lambda^\mu$ et là... on est bloqué, il nous faut comparer $\lambda^\mu$ à $\kappa$ et on est bien embêté.

    Remarque cependant que $2^\mu\leq \lambda^\mu$ donc si $\kappa^{<\kappa}\leq \kappa$, il s'ensuit que $\kappa$ est fortement limite. Donc si on avait des trucs genre régularité, on ne sera pas loin de l'inaccessibilité.

    Certainement inaccessible suffit car alors si $\lambda, \mu <\kappa$, on a $\lambda^\mu \leq 2^{\lambda \times \mu} <\kappa$ et donc voili voilou. C'est donc une condition entre fortement limite et inaccessible. J'avoue que je ne sais pas si ça implique l'inaccessibilité.

    (j'ai eu le temps ici car c'était court, je répondrai à l'autre fil plus tard )
  • Max, je crois que tu es allé un peu trop vite en affirmant que $\kappa$ est fortement limite. (C'est de ma faute, je ne te laisse jamais 5 minutes pour faire ce que tu as à faire).
    Ton raisonnement à la ligne 3 te permet seulement d'affirmer que $2^{\mu} \leq \kappa$.

    Par exemple sous HGC tu as que si $\kappa$ est régulier, alors $\lambda < \kappa \Rightarrow \kappa^{\lambda}= \kappa$, donc aussi $\kappa^{< \kappa} = \kappa$.

    D'ailleurs c'est ça qui m'a foutu dedans : j'avais lu dans un cours un truc du genre : "il est clair que $\kappa^{< \kappa}= \kappa$ puisque $\kappa$ est régulier. Et dans ma tête j'en avais fait une généralité.
    Problème : c'était dans un cours sur le forcing, et comme tu le sais ça ne coûte rien de supposer que le ground model satisfait HGC.
    D'où ma méprise.

    Conclusion : la propriété $\kappa^{<\kappa} = \kappa$ est située quelque part en dessous de inaccessible. On pourrait appeler ça un cardinal semi-inaccessible.
  • Martial : tu as raison, je m'étais fait un film (comme quoi si on avait $\leq$ on pouvait en déduire $<$ en prenant quelqu'un de plus grand, mais il se pourrait tout bêtement qu'on ait un saut de la fonction $2^\bullet$ en $\kappa$)
  • Source : K. J. Devlin, Constructibility.111752
    Le chat ouvrit les yeux, le soleil y entra. Le chat ferma les yeux, le soleil y resta. Voilà pourquoi le soir, quand le chat se réveille, j'aperçois dans le noir deux morceaux de soleil. (Maurice Carême).
  • Ah effectivement il semblerait que ça implique régulier : $\kappa^{cof(\kappa)}>\kappa$ ne demande rien, si ? C'est juste le théorème de König si je ne m'abuse

    Donc c'est quelque part entre régulier et inaccessible, mais GCH le fait collapser à régulier, c'est donc vachement en dessous d'inaccessible.

    Peut-on prouver (sans supposer GCH) qu'il en existe ? J'imagine que non

    Ce qui est amusant c'est qu'on imaginerait que GCH au contraire transformerait cette condition en "inaccessible", mais comme on a le droit à $\leq$, au contraire GCH facilite leur existence.
  • @Tous deux : Dans toute la suite je note $H_{\kappa}$ l'hypothèse $\kappa^{<\kappa} = \kappa$.
    Merci Thierry pour ce lemme 5.10, maintenant on sait quelle est la force de $H_{\kappa}$.

    Le plus pire (comme dit un de mes potes), c'est que toutes ces choses sont écrites en toutes lettres dans mon cours : chap 20, pages 32 et suivantes. Je viens de m'en rendre compte, c'est vous dire à quel point j'ai l'esprit clair en ce moment...

    Max a sûrement raison, je vois mal comment on pourrait démontrer dans ZFC qu'il existe un cardinal $\kappa$ qui vérifie $H_{\kappa}$. (A part $\aleph_0$, évidemment).
    C'est donc que l'axiome $\exists \kappa \geq \aleph_1, H_{\kappa}$ est une sorte d'espèce de genre de style d'hypothèse de forte cardinalité non encore répertoriée, lol.
  • @tous: la lettre majuscule H me rappelle, Martial, qu'il me semble qu'on note $H_a$ "officiellement" le $a$ ième cardinal qui vérifie ta propriété (sauf erreur). Donc peut-être pourrais-tu trouver un autre sigle ou la garder dans le sens que je viens de dire. Enfin, bon, dans ce fil tout le monde se comprend je crois.

    Attention: être inaccessible c'est "être un H" PLUSSSSSSSSSSS être stable par $P$. La stabilité par P ajoute beaucoup.
    Aide les autres comme toi-même car ils sont toi, ils sont vraiment toi
  • @Christophe : merci pour cette précision. Ils ont un nom, ces $H_a$ ?

    @Tous : je viens de réaliser qu'en fait dans mon cours je démontre moins que Devlin. Je suppose HGC pour prouver que $\lambda < cf(\kappa) \Rightarrow \kappa^{\lambda}= \kappa$.

    Du coup, Thierry, au risque d'abuser, pourrais-tu nous scanner le lemme 5.9 et sa preuve ?
  • @Martial. Je ne crois pas, je ne sais pas trop. Pour être tout à fait franc je me base sur un truc précis. Le discours de Woodin disant en gros:

    " on a complètement élucidé $H_\omega$ et on va maintenant élucider $H_{\omega_1}$ avec DP "

    sachant qu'ici, il désigne par $H_a$ le premier cardinal $k$ tel que $k^{<k} =k$ ET $a\leq k$. Donc ce que j'ai dit avec "a ième" est assez douteux déjà.
    Aide les autres comme toi-même car ils sont toi, ils sont vraiment toi
  • D'ailleurs, je parle de cardinaux, mais j'ai "encore tort". Je dirais qu'il désigne par $H_s$, le plus petit ensemble $E$ tel que (1) et (2) avec:

    (1) $s\subset E$

    (2) Toute partie de $E$ ayant un strictement plus petit cardinal que celui de $E$ est un élément de $E$
    Aide les autres comme toi-même car ils sont toi, ils sont vraiment toi
  • @Poirot : merci pour le doc.

    @Christophe : ce que Woodin appelle $H_{\kappa}$ dans cette histoire, ce n'est pas du tout un cardinal. C'est simplement l'ensemble de tous les ensembles qui sont "héréditairement de cardinal $\kappa$" ou, ce qui revient au même, dont la clôture transitive a une taille $< \kappa$.

    Ainsi, $H_{\omega} = V_{\omega}$, et $H_{\omega_1}$ c'est le monde où "tout est dénombrable", et c'est aussi le premier argument qui tombe sous la main pour prouver que l'axiome des parties n'est pas redondant avec le reste de ZFC.

    Je me souviens d'avoir lu un papier de Woodin où il disait en gros : "la théorie de $H_{\omega}$ c'est le taf des arithméticiens, la théorie de $H_{\omega_1}$ est pour ainsi dire torchée si on impose DP, et il va maintenant falloir s'attaquer à $H_{\omega_2}$ pour y voir plus la clair, avec la $\Omega$-logique, la $\Omega$-conjecture et toute l'artillerie lourde qui va avec".
    Il y a aussi un papier de Patrick Dehornoy : "Progrès récents sur l'hypothèse du continu", qui est une sorte de vulgarisation avancée des travaux de Woodin.
  • oalala Martial, tu as parfaitement raison:

    1/ j'ai prétendu A TORT que c'était le plus petit X tel que $a\subset X$ et $X^{<card(X)} \subset X$

    2/ Alors que c'est le plus petit X tel que $a\subset X$ et $X^{<a} \subset X$

    Sorryyyyyyy
    Aide les autres comme toi-même car ils sont toi, ils sont vraiment toi
Connectez-vous ou Inscrivez-vous pour répondre.